Ah, 3 Consecutive Primes make my day!

Can 2 , 3 , 5 \sqrt2, \sqrt3, \sqrt5 all be numbers in an arithmetic progression ?

Note: The three numbers do not need to be consecutive in the progression.

Yes No

This section requires Javascript.
You are seeing this because something didn't load right. We suggest you, (a) try refreshing the page, (b) enabling javascript if it is disabled on your browser and, finally, (c) loading the non-javascript version of this page . We're sorry about the hassle.

11 solutions

By contradiction, suppose 2 = a + n d , 3 = a + m d , 5 = a + r d \sqrt{2} = a + nd, \sqrt{3} = a + md, \sqrt{5} = a + rd for some reals a , d a,d and integers n , m , r n,m,r . Then

3 2 = ( m n ) d m n = 3 2 d \sqrt{3} - \sqrt{2} = (m - n)d \Longrightarrow m - n = \dfrac{\sqrt{3} - \sqrt{2}}{d} and 5 3 = ( r m ) d r m = 5 3 d \sqrt{5} - \sqrt{3} = (r - m)d \Longrightarrow r - m = \dfrac{\sqrt{5} - \sqrt{3}}{d} , so

r m m n = 5 3 3 2 = ( 5 3 ) ( 3 + 2 ) = 15 + 10 6 3 \dfrac{r - m}{m - n} = \dfrac{\sqrt{5} - \sqrt{3}}{\sqrt{3} - \sqrt{2}} = (\sqrt{5} - \sqrt{3})(\sqrt{3} + \sqrt{2}) = \sqrt{15} + \sqrt{10} - \sqrt{6} - 3 .

But as n , m , r n,m,r are integers, r m m n \dfrac{r - m}{m - n} must be rational, yet 15 + 10 6 3 \sqrt{15} + \sqrt{10} - \sqrt{6} - 3 is irrational. So by contradiction, 2 , 3 \sqrt{2}, \sqrt{3} and 5 \sqrt{5} cannot be elements of the same arithmetic sequence.

Added proof of the irrationality of 15 + 10 6 3 \sqrt{15} + \sqrt{10} - \sqrt{6} - 3 : As the difference between an irrational and a rational number is irrational, if suffices to prove that 15 + 10 6 \sqrt{15} + \sqrt{10} - \sqrt{6} is irrational. By contradiction, suppose that 15 + 10 6 = a \sqrt{15} + \sqrt{10} - \sqrt{6} = a for some rational number a a . Then 15 + 10 = a + 6 \sqrt{15} + \sqrt{10} = a + \sqrt{6} , and upon squaring both sides we find that

15 + 10 + 2 150 = a 2 + 6 + 2 a 6 19 a 2 = 2 a 6 2 25 × 6 = ( 2 a 10 ) 6 15 + 10 + 2\sqrt{150} = a^{2} + 6 + 2a\sqrt{6} \Longrightarrow 19 - a^{2} = 2a\sqrt{6} - 2\sqrt{25 \times 6} = (2a - 10)\sqrt{6} .

Now 19 a 2 19 - a^{2} is rational, as is 2 a 10 2a - 10 , but since 6 \sqrt{6} is irrational ( 2 a 10 ) 6 (2a - 10)\sqrt{6} is also irrational, (being the product of a non-zero rational and an irrational number), thus providing the desired contradiction. (Note that 15 + 10 7 \sqrt{15} + \sqrt{10} \approx 7 and 6 2.4 \sqrt{6} \approx 2.4 , so a 4.6 a \approx 4.6 and thus 2 a 10 0 2a - 10 \ne 0 , satisfying the non-zero condition.)

Can you prove that those three numbers cannot be in G.P too.

Ram Mohith - 2 years, 10 months ago

Log in to reply

Yes, again by contradiction. Suppose 2 = a r m , 3 = a r n \sqrt{2} = ar^{m}, \sqrt{3} = ar^{n} and 5 = a r t \sqrt{5} = ar^{t} for distinct positive integers m , n , t m,n,t and non-zero reals a , r a,r . Then

3 2 = r n m 3 2 = r 2 ( n m ) r = ( 2 3 ) 1 2 ( n m ) \dfrac{\sqrt{3}}{\sqrt{2}} = r^{n - m} \Longrightarrow \dfrac{3}{2} = r^{2(n - m)} \Longrightarrow r = \left(\dfrac{2}{3}\right)^{\frac{1}{2(n - m)}} and

5 3 = r t n 5 3 = r 2 ( t n ) r = ( 5 3 ) 1 2 ( t n ) \dfrac{\sqrt{5}}{\sqrt{3}} = r^{t - n} \Longrightarrow \dfrac{5}{3} = r^{2(t - n)} \Longrightarrow r = \left(\dfrac{5}{3}\right)^{\frac{1}{2(t - n)}} .

After equating these two expressions for r r and simplifying we will end up with 2 t n 5 n m = 3 t m 2^{t - n}5^{n - m} = 3^{t - m} . But by the Fundamental Theorem of Arithmetic we cannot have two distinct prime factorizations of an integer, completing the proof by contradiction.

Brian Charlesworth - 2 years, 10 months ago

Log in to reply

Yes it is absolutely the correct procedure

Ram Mohith - 2 years, 10 months ago

Absolutely correct.Good Job!!!

D K - 2 years, 10 months ago

I had to look up the definition of an arithmetic progression. Then I used innate sense of cardinality as applied to the number of primes I can count mentally or using two hands etc and intuited that the roots of 2, 3, and 5 'felt' like they were irrational or 'unboildownable' and concluded that they couldn't represent part of a sequence that moved by a constant amount.

A Former Brilliant Member - 2 years, 10 months ago

I'm afraid I am not understanding how did you get the 3*sqrt(3). Could you explain it? I get sqrt(15)-sqrt(6).

P S - 2 years, 10 months ago

Log in to reply

You are right; thanks for catching that. I'm surprised no one had noticed that mistake before now.

Brian Charlesworth - 2 years, 10 months ago

Don't you need to prove that the final number is irrational?

Malcolm Rich - 2 years, 10 months ago

Log in to reply

Good point. Subtracting a rational from an irrational yields an irrational, so it suffices to prove that 15 + 10 6 \sqrt{15} + \sqrt{10} - \sqrt{6} is irrational. By contradiction, suppose this expression equals some rational a a . Then 15 + 10 = a + 6 \sqrt{15} + \sqrt{10} = a + \sqrt{6} . Squaring both sides yields

15 + 10 + 2 150 = a 2 + 6 + 2 a 6 19 a 2 = 2 a 6 2 6 × 25 = ( 2 a 10 ) 6 15 + 10 + 2\sqrt{150} = a^{2} + 6 + 2a\sqrt{6} \Longrightarrow 19 - a^{2} = 2a\sqrt{6} - 2\sqrt{6 \times 25} = (2a - 10)\sqrt{6} .

Now 19 a 2 19 - a^{2} is rational, as is 2 a 10 2a - 10 , but since 6 \sqrt{6} is irrational the product ( 2 a 10 ) 6 (2a - 10)\sqrt{6} is irrational, giving us the desired contradiction.

Note: The product of a non-zero rational r 0 r \ne 0 and an irrational x x is irrational. By contradiction, suppose r x = q rx = q for some non-zero rational q q . Then x = q r x = \dfrac{q}{r} would be rational by closure of rationals over (non-zero) division, giving us our contradiction. Finally, since 15 + 10 6 \sqrt{15} + \sqrt{10} \approx 6 and 6 2.4 \sqrt{6} \approx 2.4 we know that a 5 2 a 10 0 a \ne 5 \Longrightarrow 2a - 10 \ne 0 , so the non-zero condition is satisfied.

Hmmm.... This "clarification" was as long as my posted solution. I'm glad you asked; I had to think about that for a while. :)

Brian Charlesworth - 2 years, 10 months ago

Log in to reply

Cheers. I'd reached the same stage and assumed the value was irrational because it "looked" irrational. It was a minor challenge but as a proof it looks complete now.

Malcolm Rich - 2 years, 10 months ago

I think there is a typo in the first line : should it not say root(5) = a + rd ?

Sevan Stephan - 2 years, 10 months ago

Log in to reply

You're right; thanks for catching that. :)

Brian Charlesworth - 2 years, 10 months ago
Jason Dyer Staff
Jul 24, 2018

For this proof we're going to use the facts that

addition / subtraction / multiplication / division in nonzero rationals is closed

and

the square roots of positive nonsquare integers (2, 3, 5, 6, etc.) are irrational.

Suppose 2 , \sqrt{2}, 3 , \sqrt{3}, and 5 \sqrt{5} are all part of the same arithmetic sequence. Then there must be integers a a and b b such that

a ( 3 2 ) = b ( 5 3 ) a(\sqrt{3}-\sqrt{2}) = b(\sqrt{5}-\sqrt{3})

Letting rational number R = a b R = \frac{a}{b} we can write this as

R ( 3 2 ) = 5 3 R 3 R 2 = 5 3 \begin{aligned} R(\sqrt{3}-\sqrt{2}) &= \sqrt{5}-\sqrt{3} \\ R\sqrt{3}-R\sqrt{2} &= \sqrt{5}-\sqrt{3} \end{aligned}

Add 3 \sqrt{3} to both sides. This gets the term ( R + 1 ) 3 (R+1)\sqrt{3} where R + 1 R+1 is rational. Let Q = R + 1. Q=R+1.

Q 3 R 2 = 5 Q\sqrt{3}-R\sqrt{2} = \sqrt{5}

Square both sides.

3 Q 2 + 2 R 2 2 Q R 6 = 5 3Q^2 + 2R^2 - 2QR\sqrt{6} = 5

Now we can subtract 3 Q 2 3Q^2 and 2 R 2 2R^2 from 5 5 and the result must be rational.

2 Q R 6 = ( a rational number ) -2QR\sqrt{6} = (\text{a rational number})

We can also then divide both sides by 2 Q R -2QR and the right hand of the equal sign will still be rational.

6 = ( a rational number ) \sqrt{6} = (\text{a rational number})

But since the square root of a nonsquare integer is irrational, this is a contradiction. Hence 2 , \sqrt{2}, 3 , \sqrt{3}, and 5 \sqrt{5} must not be part of the same arithmetic sequence. \square

Moderator note:

There's many proofs that if N N is not a perfect square then N \sqrt{N} is not rational. Here is one of the simplest:

Suppose N \sqrt{N} is a rational number B A \frac{B}{A} in lowest terms.

With some algebra ...

N = B A N N = B A N N A B = N \begin{aligned} \sqrt{N} &= \frac{B}{A} \\ \sqrt{N}\sqrt{N} &= \frac{B}{A}\sqrt{N} \\ \frac{NA}{B} &= \sqrt{N} \end{aligned}

... we can justify that B A = N A B . \frac{B}{A} = \frac{NA}{B} .

Since B A \frac{B}{A} is in lowest terms, there exists an integer c c such that B c = N A Bc = NA and A c = B . Ac = B .

From A c = B Ac = B we also have c = B A . c = \frac{B}{A} . This means B A \frac{B}{A} is an integer, implying N \sqrt{N} is an integer and consequently N N is a perfect square.

Therefore, if N \sqrt{N} is a rational number, N N is a perfect square.

This also indicates the contrapositive: if N N is not a perfect square, then N \sqrt{N} is not a rational number.

I love the proof and that makes a lot of sense. I arrived at the correct answer, but possibly for an invalid reason and I would appreciate a critique of the approach -- i.e. is there a flaw in my logic?

sqrt(2), sqrt(3), and sqrt(5) are all irrational. They're obviously not sequential terms in an arithmetic sequence, but the rules stated that there are possibly unlisted terms between these. As these are irrational numbers, there is no way to define intermediate terms that would satisfy this.

That's it. That's my logic. Is it flawed? Another way of asking this: Do three irrational numbers exist that do not share a common factor yet are part of the same arithmetic sequence?

Michael Carmen Maguire - 2 years, 10 months ago

Log in to reply

Sure: pi, pi + e, pi + 2e work (these are consecutive, even).

If you want the difference to be an integer, pi, pi+1, and pi+2 are also irrational.

Jason Dyer Staff - 2 years, 10 months ago

There's no way to contradict this explanation. I get it. But I can't see why my logic is false: let's say all three numbers (√2; √3 and √5) are points in a line (because they are!) And the question is: is there a value that we can add to the first point, arrives exactly at the second point and ends exactly at the third point? I think yes. Because I can pick a infinitely small number. Where did I went wrong with this?

Anlev Joel - 2 years, 10 months ago

Log in to reply

You can't just say "I pick an infinitely small number"; numbers in the real number system are actual points you can find on the line, so you actually have to name a number. (This is why in calculus we don't just pick some arbitrary variable to be infinitely small, but have to go through finagling with limits and say something like x when the limit of x goes to 0.)

Jason Dyer Staff - 2 years, 9 months ago

Assuming that 2 \sqrt 2 , 3 \sqrt 3 and 5 \sqrt 5 are number of an arithmetic progression with a common difference d d . We can assume that d > 0 d>0 without loss of generality. Then the following system of equations is true.

{ 2 + m d = 3 . . . ( 1 ) 3 + n d = 5 . . . ( 2 ) \begin{cases} \sqrt 2 + md = \sqrt 3 & ...(1) \\ \sqrt 3 + nd = \sqrt 5 & ...(2) \end{cases} , where m m and n n are positive integers.

Then we have d = 3 2 m = 5 3 n d = \dfrac {\sqrt 3-\sqrt 2}m = \dfrac {\sqrt 5-\sqrt 3}n and that:

5 3 3 2 = n m ( 5 3 ) ( 3 + 2 ) = n m 15 3 + 10 6 = n m \begin{aligned} \frac {\sqrt 5-\sqrt 3}{\sqrt 3-\sqrt 2} & = \frac nm \\ (\sqrt 5-\sqrt 3)(\sqrt 3+\sqrt 2) & = \frac nm \\ \sqrt {15} -3 + \sqrt{10} - \sqrt 6 & = \frac nm \end{aligned}

Since LHS is irrational and RHS is rational, there is no solution to the above equation and therefore the assumption is false . No , 2 \sqrt 2 , 3 \sqrt 3 and 5 \sqrt 5 are not numbers of an arithmetic progression.

Mikhail Zyatin
Aug 2, 2018

Let's recall the formula for the arithmetic progression: x i = a + b i x_i=a + b*i . Which means that if we interpret this formula as a function of i , then all members of the progression are equally spaced laying on the straight line, where a is the vertical shift and b is the slope of this line.

2 \sqrt{2} , 3 \sqrt{3} and 5 \sqrt{5} are the values of the function y = x y=\sqrt{x} . So, the question is, is it possible to intersect parabola with the straight line in 3 points. The answer is obviously not.

I feel that this proof is somewhat informal. There are other solutions here which are much more logically consistent then mine but they are based on a different ideas. So, I am wondering, how this idea can be turned into a solid mathematical proof.

Seems solid to me

tom callahan - 2 years, 10 months ago

Given f ( x ) = m x + b f(x) = mx + b and g ( x ) = x g(x) = \sqrt{x} , try proving that f and g do not contain more than 2 intersections. Intersections appear where f ( x ) = g ( x ) f(x) = g(x) .

f ( x ) = g ( x ) f(x) = g(x)

m x + b = x mx + b = \sqrt{x}

( m x ) 2 + b = x (mx)^2 + b = x // square both sides

Equation is quadratic. Maximum number of solutions is 2.

Hmm... I don't know though. What I wrote wasn't very self-satisfying, in my opinion. Hope it quells some doubts though.

John L - 2 years, 10 months ago

Log in to reply

Thank you, Johnathan!

That's what I feel when it comes to geometrical intuitions. They are very powerful and satisfying, but also sometimes I can't convince myself that I've translated the problem to geometry in a right way.

Mikhail Zyatin - 2 years, 10 months ago

If 2 \sqrt{2} , 3 \sqrt{3} , 5 \sqrt{5} is arithmetic progression, then must 3 2 5 3 \frac { \sqrt {3} - \sqrt {2} } { \sqrt {5} - \sqrt {3} } = 1

( 3 2 ) ( 5 + 3 ) ( \sqrt {3} - \sqrt {2} ) ( \sqrt {5} + \sqrt {3} ) = 2

15 + 1 \sqrt {15} + 1 = 10 + 6 \sqrt {10} + \sqrt {6}

16 + 2 15 \sqrt {15} = 16 + 2 60 2 \sqrt {60}

15 = 60 \sqrt {15} = \sqrt {60} , this is wrong.

So, 2 , 3 , 5 \sqrt {2} , \sqrt {3} , \sqrt {5} not an arithmetic progression.

The terms need not be consecutive so that initial fraction doesn't have to be equal to 1

Tomás Carvalho - 2 years, 10 months ago
Michael Mendrin
Jul 29, 2018

If we let

a 1.0461659 a \approx 1.0461659
d 0.462547 d \approx 0.462547
r 0.937364 r \approx 0.937364

Then

( a + d ) r 2 (a+d)r \approx \sqrt{2}
( a + 2 d ) r 2 3 (a+2d)r^2 \approx \sqrt{3}
( a + 4 d ) r 4 5 (a+4d)r^4 \approx \sqrt{5}

hmm...

Note: The form is ( a + d n ) r n (a+dn)r^n where n n is a positive integer, while a , d a, d both can be irrational. For this problem, r = 1 r=1 , for which there is no solution.

What detail is that?

Pi Han Goh - 2 years, 10 months ago

Log in to reply

Made a mistake with a more general arithmetico-geometric sequence. See my revised note.

Michael Mendrin - 2 years, 10 months ago

Log in to reply

a , d , r a,d,r are all rational numbers, then the latter 3 equations cannot be true.

Pi Han Goh - 2 years, 10 months ago

Log in to reply

@Pi Han Goh If we let r = 1 r=1 , then we have the problem as posted. Obviously no linear combination of rational numbers can come up with irrational square roots, but we're assuming that a , d a, d can be irrational. But if r = 1 r=1 , then no solution exists.

Michael Mendrin - 2 years, 10 months ago

Log in to reply

@Michael Mendrin If you are assuming that a , d a,d are rational, then why did you write (the values of) a a and d d as rational numbers?

Pi Han Goh - 2 years, 10 months ago

Log in to reply

@Pi Han Goh What I am saying is that, for example, if a , d a, d are both rational, obviously a + d a+d cannot ever equal 2 \sqrt{2} . This posted problem asks if 2 , 3 , 5 \sqrt{2},\sqrt{3}, \sqrt{5} could be in terms in any arithmetic sequence, so at least a , d a, d both cannot be rational. But even if they both are not rational, a solution still doesn't exist, and we see multiple proofs of it here. But if we include powers of r r , where r r can be irrational also, then a solution does exist.

The only thing that must be not only rational, but a positive integer, is n n

Michael Mendrin - 2 years, 10 months ago

Log in to reply

@Michael Mendrin Oh, so your first equations in your solution should be:

a 1.0461659 a\approx1.0461659
d 0.462547 d\approx 0.462547
r 0.937364 r \approx 0.937364

Okay then.

Pi Han Goh - 2 years, 10 months ago

Log in to reply

@Pi Han Goh fixed, see revised notes

Michael Mendrin - 2 years, 10 months ago

So you're saying that, while they cannot be in AP or GP, (as I showed in a comment to my posted solution), they can be in AGP. That's a curious result ....

Brian Charlesworth - 2 years, 10 months ago

Log in to reply

If there are 3 unknowns and 3 equations, you'd think there'd be a solution, but it turns out that for n = 1 , 2 , 3 n=1,2,3 , there isn't one. That's the hiccup that I found curious.

Michael Mendrin - 2 years, 10 months ago
Andrew Wee
Aug 3, 2018

This is similar to saying that: Can 2^2, 3^2 and 5^2 all be numbers in Arithmetic Progession?

Basically, anything to do with manupilating powers of a number will increase and decrease depending on the number.

Eg. 1^2=1 2^2=4 3^2=9 4^2=16 5^2=25

And on an on and on... No common difference

Ervyn Manuyag
Jul 31, 2018

They all need to be whole numbers?

Ram 1537
Jul 31, 2018

Can’t we just assume that they can’t constitute three numbers in an arithmetic series because the irrationals are densed along the real axis therefore there’s no foreseenable constant gap between them?

Vinod Kumar
Jul 30, 2018

As {(√3-√2)/(√5-√3)} is irrational, it can't be in AP.

0 pending reports

×

Problem Loading...

Note Loading...

Set Loading...